PT38.S4.Q13-- a recent study reveals

DarklordDarklord Alum Member

Hi,

Can anyone explain why B here is 100% wrong? I thought that B could weaken the argument because it could state the conclusion of the study is wrong.

Thanks!
Any #help would be appreciated!

Comments

  • Confidence150Confidence150 Alum Member
    1417 karma

    We are trying to figure out whether or not television advertising affects a children’s preference for breakfast cereals.

    The phrase “can be influenced” in answer choice B doesn’t strengthen or weaken with a bit of a neutral tone.

    The key phrase in answer choice A “are influenced by” is a key phrase with a more definite tone.

  • DarklordDarklord Alum Member
    586 karma

    Hmm ok thanks @Magnificent2021

Sign In or Register to comment.